Difference between revisions of "2025 AIME II Problems/Problem 12"
(→Solution 1) |
m (→Problem) |
||
(4 intermediate revisions by 2 users not shown) | |||
Line 4: | Line 4: | ||
• The area of <math>A_iA_1A_{i+1}</math> is <math>1</math> for each <math>2 \le i \le 10</math>, | • The area of <math>A_iA_1A_{i+1}</math> is <math>1</math> for each <math>2 \le i \le 10</math>, | ||
+ | |||
• <math>\cos(\angle A_iA_1A_{i+1})=\frac{12}{13}</math> for each <math>2 \le i \le 10</math>, | • <math>\cos(\angle A_iA_1A_{i+1})=\frac{12}{13}</math> for each <math>2 \le i \le 10</math>, | ||
+ | |||
• The perimeter of <math>A_1A_2\dots A_{11}</math> is <math>20</math>. | • The perimeter of <math>A_1A_2\dots A_{11}</math> is <math>20</math>. | ||
Line 10: | Line 12: | ||
== Solution 1== | == Solution 1== | ||
+ | Set <math>A_1A_2 = x</math> and <math>A_1A_3 = y</math>. By the first condition, we have <math>\frac{1}{2}xy\sin\theta = 1</math>, where <math>\theta = \angle A_2 A_1 A_3</math>. Since <math>\cos\theta = \frac{12}{13}</math>, we have <math>\sin\theta = \frac{5}{13}</math>, so <math>xy = \frac{26}{5}</math>. Repeating this process for <math>\triangle A_i A_1 A_{i+1}</math>, we get <math>A_1A_2 = A_1A_4 = \ldots A_1A_{10} = x</math> and <math>A_1A_3 = A_1A_5 = \ldots A_1A_{11} = y</math>. Since the included angle of these <math>9</math> triangles is <math>\theta</math>, the square of the third side is | ||
+ | <cmath>x^2 + y^2 - 2xy\cos\theta = x^2 + y^2 - \frac{52}{5}\cdot \frac{12}{13} = x^2 + y^2 - \frac{48}{5} = (x+y)^2 - 20.</cmath> | ||
+ | Thus the third side has length <math>\sqrt{(x+y)^2 - 20}.</math> The perimeter is constructed from <math>9</math> of these lengths, plus <math>A_{11}A_1 + A_1A_2 = x + y</math>, so <math>9\sqrt{(x+y)^2 - 20} + x + y = 20</math>. We seek the value of <math>x + y,</math> so let <math>x + y = a</math> so | ||
+ | <cmath> | ||
+ | \begin{align*} | ||
+ | 9\sqrt{a^2 - 20} + a &= 20\\ | ||
+ | 81(a^2 - 20) &= 400 - 40a + a^2\\ | ||
+ | 4a^2 + 2a - 101 &= 0 \\ | ||
+ | a &= \frac{-2 \pm \sqrt{1620}}{8} = \frac{-1 \pm \sqrt{405}}{4} = \frac{-1 \pm 9\sqrt{5}}{4}. | ||
+ | \end{align*} | ||
+ | </cmath> | ||
+ | Taking the positive solution gives <math>m + n + p + q = 1 + 9 + 5 + 4 = \boxed{\textbf{(019)}}.</math> | ||
− | + | -Benedict T (countmath1) | |
− | |||
− | |||
− | + | ==See also== | |
+ | {{AIME box|year=2025|num-b=11|num-a=13|n=II}} | ||
− | + | {{MAA Notice}} |
Latest revision as of 22:03, 15 February 2025
Problem
Let be a non-convex
-gon such that
• The area of is
for each
,
• for each
,
• The perimeter of is
.
If can be expressed as
for positive integers
with
squarefree and
, find
.
Solution 1
Set and
. By the first condition, we have
, where
. Since
, we have
, so
. Repeating this process for
, we get
and
. Since the included angle of these
triangles is
, the square of the third side is
Thus the third side has length
The perimeter is constructed from
of these lengths, plus
, so
. We seek the value of
so let
so
Taking the positive solution gives
-Benedict T (countmath1)
See also
2025 AIME II (Problems • Answer Key • Resources) | ||
Preceded by Problem 11 |
Followed by Problem 13 | |
1 • 2 • 3 • 4 • 5 • 6 • 7 • 8 • 9 • 10 • 11 • 12 • 13 • 14 • 15 | ||
All AIME Problems and Solutions |
The problems on this page are copyrighted by the Mathematical Association of America's American Mathematics Competitions.